Find the missing side of each triangle. ​

 Find The Missing Side Of Each Triangle.

Answers

Answer 1

Answer:

5) 12.2       6) 2

Step-by-step explanation:

Use Pythagoras for both

Rearrange for x in question 6


Related Questions

When a homeowner has a 25-year variable-rate mortgage loan, the monthly payment R is a function of the amount of the loan A and the current interest rate i (as a percent); that is, R = f(A). Interpret each of the following. (a) R140,000, 7) - 776.89 For a loan of $140,000 at 7% interest, the monthly payment is $776.89. For a loan of $140,000 at 7.7689% interest, 700 monthly payments would be required to pay off the loan. For a loan of $140,000 at 7% interest, 776.89 monthly payments would be required to pay off the loan. For a loan of $140,000 at 7.7689% interest, the monthly payment is $700.

Answers

The monthly payment required to pay off a loan of $140,000 at 7% interest would be $776.89 is the correct statement(A).

The statement given is describing a function that relates the monthly payment R of a 25-year variable-rate mortgage loan to the loan amount A and the current interest rate i.

The given values are R = $776.89 and A = $140,000, with an interest rate of 7%. This means that the monthly payment required to pay off a loan of $140,000 at 7% interest would be $776.89.

However, the other statements are incorrect interpretations. For instance, the statement "For a loan of $140,000 at 7.7689% interest, 700 monthly payments would be required to pay off the loan" is incorrect.

This is because the number of payments required to pay off a loan depends not only on the loan amount and interest rate, but also on the term of the loan.

Similarly, the statement "For a loan of $140,000 at 7% interest, 776.89 monthly payments would be required to pay off the loan" is also incorrect, as the number of payments required would be determined by the term of the loan.

Finally, the statement "For a loan of $140,000 at 7.7689% interest, the monthly payment is $700" is also incorrect. This is because, for the given loan amount and interest rate, the monthly payment required would be $776.89, as calculated above.

For more questions like Interest click the link below:

https://brainly.com/question/13324776

#SPJ11

Which graph is the result of reflecting f(x) = 1/4(8) across the y-axis and then across the x-axis?
O
B
7
6
5
->
32
Shy
-8-7-6-5-4-3-2-1₁ 12 x

Answers

The graph οf g(x) is the same as its reflectiοn acrοss bοth the y-axis and the x-axis.

What is a graph?

A graph is a structure amοunting tο a set οf οbjects in which sοme pairs οf the οbjects are in sοme sense "related". The οbjects cοrrespοnd tο mathematical abstractiοns called vertices and each οf the related pairs οf vertices is called an edge.

Tο reflect a functiοn acrοss the y-axis, we replace x with -x in the functiοn. This gives us:

f(-x) = 1/4(8) = 2

This functiοn represents the reflectiοn οf f(x) acrοss the y-axis. Tο reflect this functiοn acrοss the x-axis, we replace f(-x) with -f(-x). This gives us:

-f(-x) = -2

Therefοre, the functiοn that results frοm reflecting f(x) = 1/4(8) acrοss the y-axis and then acrοss the x-axis is:

g(x) = -2

This is a hοrizοntal line that intersects the y-axis at -2. The graph οf g(x) is a straight line parallel tο the x-axis, and it dοes nοt change when reflected acrοss the y-axis οr x-axis. Sο, the graph οf g(x) is the same as its reflectiοn acrοss bοth the y-axis and the x-axis.

Learn mοre abοut graphs οn:

https://brainly.com/question/17267403

#SPJ1

Can someone help me to work this out???????????????????????

Answers

Answer:

80 degrees

Step-by-step explanation:

Remember that corresponding angles are equal!

angle CBD and angle BFH are obviously corresponding angles.

(SEE ATTACHMENT IF YOU DON'T UNDERSTAND)

So, angle CBD = angle BFH = 100 degrees.

The angles of a line add to 180 degrees

angle BFH is 100 degrees and one part of this line. So the other part (aka angle x) is 180 - 100 = 80 degrees.

Thus, x is 80 degrees.

Write down two factors of 24 that are primenumber

Answers

the prime factors of 24 are 2 and 3, which combine to give the unique prime factorization of 24 as 2^3 × 3.

There are no factors of 24 that are prime numbers. A factor of a number is a whole number that divides that number without leaving a remainder. Prime numbers, on the other hand, are numbers that are divisible only by 1 and themselves, and cannot be expressed as the product of any other numbers.

The prime factors of 24 are 2, 2, and 3. We can factorize 24 as 2 × 2 × 2 × 3 or 2^3 × 3. Here, 2 and 3 are both prime numbers, but they are not factors of 24 in isolation. They are only prime factors of 24 when combined in the manner shown.

This fact highlights an important concept in number theory: the uniqueness of prime factorization. Every composite number can be expressed as a unique product of prime numbers. This fundamental theorem of arithmetic is crucial in many areas of mathematics, including cryptography, where it is used to secure communications and protect sensitive information.

In summary, there are no factors of 24 that are prime numbers. However, the prime factors of 24 are 2 and 3, which combine to give the unique prime factorization of 24 as 2^3 × 3.

To know more about prime factorization click here:

brainly.com/question/29763746

#SPJ4

the length of a rectangle is 3 in longer than its width. if the perimeter of the rectangle is 50 in, find its length and widths​

Answers

First, re-read the problem until you understand it and can put it into your own words.  I re-wrote it like this:  "Find the area of a rectangle by first finding the length (L) and the width (W)."   [note that I added "find L and W," but that is how I'm going to solve the problem;  I could also have said that we will need the formulas, P=2L+2W and A=LW, but you knew that already, right?).

Translate the problem:

 "The length of a rectangle is 3 ft longer than its width"      means

         L                             =    3             +         W                 (eq1)

 "the perimeter of the rectangle is 30 ft"     means

            P                                = 50                      (eq2)

So, now the math is easy, just find L and W so we can compute the area:

     P = 50 = 2L + 2W                         (eq3; from eq2 and the formula for P)

          50 = 2(3+W)  +  2W              (use eq1 to substitute for L)

          50 = 6 + 2W   + 2W              (distribute)

           50 = 6 + 4W                       (collect terms)

            44 = 4W                        (subtract 6 from both sides)

            11 ft = W                        (divide both sides by 4)

Use the easiest equation (either eq1 or else eq3) to find L:

          L = 3 + W        (eq1)

          L = 3 + 11

         L = 14 ft

What is the area (A)?

          A = L*W

          A = (14 ft) x (11 ft)

          A = 154 sq ft

In a random sample of 200 school district residents, 94 stated they are in favor of starting the school day 15 minutes later each day. Calculate a 90% confidence interval for the true proportion of district residents who are in favor of starting the day later

Answers

The 90% confidence interval for the proportion of district residents in favor of starting the school day 15 minutes later is (0.392, 0.548). The true proportion is estimated to lie within this interval with 90% confidence.

To calculate the 90% confidence interval for the true proportion of district residents who are in favor of starting the school day 15 minutes later, we can use the following formula:

CI = p ± z*(√(p*(1-p)/n))

where:

CI: confidence interval

p: proportion of residents in favor of starting the day later

z: z- score based on the confidence level (90% in this case)

n: sample size

First, we need to calculate the sample proportion:

p = 94/200 = 0.47

Next, we need to find the z- score corresponding to the 90% confidence level. Since we want a two-tailed test, we need to find the z- score that cuts off 5% of the area in each tail of the standard normal distribution. Using a z-table, we find that the z- score is 1.645.

Substituting the values into the formula, we get:

CI = 0.47 ± 1.645*(√(0.47*(1-0.47)/200))

Simplifying this expression gives:

CI = 0.47 ± 0.078

Therefore, the 90% confidence interval for the true proportion of district residents who are in favor of starting the school day 15 minutes later is (0.392, 0.548). We can be 90% confident that the true proportion lies within this interval.

Learn more about the confidence interval here: brainly.com/question/24131141

#SPJ4

Find the total number of outcomes for picking a day of the week and a month of the year. A 84 , B 19 , C 60 , D 210

Answers

Option A is the correct option for determining the total number of possible outcomes when selecting a day of the week and a month of the year, which is 84.

To find the total number of outcomes for picking a day of the week and a month of the year, you need to multiply the number of options for each category. There are 7 days in a week, so there are 7 options for picking a day. There are 12 months in a year, so there are 12 options for picking a month. To find the total number of outcomes, you multiply the number of options for each category: 7 x 12 = 84. Therefore, the total number of outcomes for picking a day of the week and a month of the year is 84. Option A is the correct answer.

Learn more about probability here: brainly.com/question/30034780

#SPJ4

Find an expression which represents the difference when (9x-4)(9x−4) is subtracted from (7x-2)(7x−2) in simplest terms.

Answers

Answer:

To subtract (9x-4)(9x-4) from (7x-2)(7x-2), we need to expand both of them and then subtract. Using the formula (a-b)^2 = a^2 - 2ab + b^2, we get:

(7x-2)(7x-2) - (9x-4)(9x-4)
= (49x^2 - 28x + 4) - (81x^2 - 72x + 16)
= 49x^2 - 28x + 4 - 81x^2 + 72x - 16
= -32x^2 + 44x - 12

Therefore, the expression which represents the difference when (9x-4)(9x−4) is subtracted from (7x-2)(7x−2) is -32x^2 + 44x - 12.

Tristan is going to invest in an account paying an interest rate of 6.2% compounded monthly. How much would Tristan need to invest, to the nearest cent, for the value of the account to reach $18,100 in 12 years?

Answers

Answer:

Step-by-step explanation:

We can use the compound interest formula to solve this problem:

A = P(1 + r/n)^(nt)

Where:

A = Final amount (which is given as $18,100)

P = Principal amount (the amount Tristan needs to invest)

r = Annual interest rate (6.2%)

n = Number of times interest is compounded per year (12)

t = Time in years (12)

Substituting the values in the formula:

18,100 = P(1 + 0.062/12)^(12*12)

Simplifying the right-hand side:

18,100 = P(1.00516666667)^144

Dividing both sides by (1.00516666667)^144:

P = 18,100 / (1.00516666667)^144

P = $8,167.75 (rounded to the nearest cent)

Therefore, Tristan needs to invest $8,167.75, to the nearest cent, for the value of the account to reach $18,100 in 12 years.

I need help I need to show my work please help

Answers

Answer:

The 2nd equation is false.

Step-by-step explanation:

40 ≠ 58, so we can say the 2nd equation is false.

A geometric sequence has first term 5 and common ratio 2. the sum of the first n terms of the sequence is 163 835. What is n ? [Answer format: integer, no units]

Answers

First term in a geometric sequence is 5, and common ratio is 2. The sum of the first n terms of the sequence is 163,835.

The following formula can be used to calculate the sum of the first n terms of a geometric sequence:

Sn = a (1 - rn) over (1 - r),

where

a is the first term,

r is the common ratio, and

n is the number of terms.

So, Sn = a (1 - rn) over (1 - r) => 163835 = 5 (1 - 2n) over (1 - 2).

Simplifying it, we have

   163835 = 5 (1 - 2n) over (-1)

=> 163835 = -5 + 10n

=> 163840 = 10n

  => n = 16,384.

Therefore, the value of n is 16,384.

You can learn more about geometric sequence at

https://brainly.com/question/24643676

#SPJ11

If the original quantity is 225 and the quality of 320 what is the percentage increase?

Answers

Step-by-step explanation:

The change fro 225 to 320 is 95

95 is what percentage of 225?

95/225 x 100% = 42.2% increase

Need help with question 4 please

Answers

Answer:

Question 4

a) 0.667

b) 0.833

c) 0.833

d) 0

Question 5

a) 32 yards

b) $312

Step-by-step explanation:

Probability question 4

The best way to solve these probability questions is to enumerate and count the total set of outcomes and then enumerate and count the specific set of outcomes they are looking for in each part .

Since the experiment consists of rolling a fair single 6-sided die  we can enumerate the set of  possible outcomes as
{1, 2, 3, 4, 5, 6}. This is also referred to in probability as sample space.

There are 6 possible outcomes

The probability of getting any one number is 1/6 whether it be a 1, a 2 or 3, 4, 5 or 6.

In general P(a specific outcome) = Number of ways of getting that outcome ÷ total number of all possible outcomes

part a)
Rolling an even==>  Set of outcomes  = {2, 4, 6}
Rolling a 3 => {3}

Rolling a 3 or even = {2, 3, 4, 6}; 4 elements
P(even or 3 ) = 4/6 = 2/3 = 0.667

Part b)
Number > 2 ==> { 3, 4, 5, 6}
Even: {2, 4, 6}

Even or greater than 2 = {2, 3, 4, 5, 6}; 5 elements  - we do not duplicate

P(Even or greater than 2 ) = 5/6 = 0.833

part c)
Odd number: {1, 3, 5}
Number less than 6: {1, 2, 3, 4, 5}
Combining the two we get: {1, 2, 3, 4, 5} for a total of 5 outcomes

P(odd or less-than 6 ) = 5/6 = 0.833

part(d)
Rolling a 2 and a 4
You cannot roll a 2 and a 4 in one toss so the probability is 0

Question 5
I think you have overly complicated the working of this problem. There is no need to use right triangle, hypotenuse etc

If you have a rectangle with length = L and width = W, the perimeter is just

P = 2( L + W)

Here we can choose any dimension for L and the other for W
Let's choose L = 11 (greater number)
W = 5

Perimeter P = 2 (11 + 5) = 2(16) = 32 yards

b) It costs $3.25 per foot for the fencing..
Careful!
The  only tricky part here is the units. It is given in $ per square foot whereas the dimensions of the plot are given in yards and the perimeter is also in yards. Watch out for these tricks in other questions

Convert perimeter from yards to feet

32 yards = 32 x 3 = 96 feet  (there are 3 feet to a yard)

Cost of 96 feet of fencing at $3.25 per foot
= 96 x 3.25 = $312


Consider the initial value problem D'=13_), zo = [ 2] y(0) = a. Find the eigenvalue 1, an eigenvector v1, and a generalized eigenvector v2 for the coefficient matrix of this linear system. - l= , Vj = -- b. Find the most general real-valued solution to the linear system of differential equations. Use t as the independent variable in your answers. y(t) = C1 + C2 c. Solve the original initial value problem. yı(t) = ya(t) =

Answers

The solution to the initial value problem is yı(t) = ya(t) = [a e^(13t) - (e^(13t) - 1)] (1, 0).

Given information:Consider the initial value problem D' = 13_, zo = [ 2] y(0) = a.To find the eigenvalue 1, an eigenvector v1, and a generalized eigenvector v2 for the coefficient matrix of this linear system.Step-by-step explanation:The given differential equation isD' = 13_Y = [ 2] y(0) = a.The coefficient matrix A for this system is:A = [13]The eigenvalues λ1, λ2 are obtained by solving the characteristic equation:det(A - λI) = 0where I is the 2x2 identity matrixdet(A - λI) = (13 - λ)(-λ) - 2(0) = λ(λ - 13)This equation has two roots:λ1 = 0, λ2 = 13.The corresponding eigenvectors v1 and v2 are obtained by solving the equations:(A - λ1I) v1 = 0, (A - λ2I) v2 = 0. For λ1 = 0, we have(A - λ1I) v1 = (13 0) (v1[1])= (0 0) (v1[2])which implies that v1[1] = 0 and v1[2] is free. Therefore, v1 = (0, 1). For λ2 = 13, we have(A - λ2I) v2 = (0 0) (v2[1])= (0 -13) (v2[2])which implies that v2[1] is free and v2[2] = 0. Therefore, v2 = (1, 0).The generalized eigenvector v3 for λ1 = 0 is obtained by solving the equation:(A - λ1I) v3 = v2For this equation, we have(13 0) (v3[1])= (0 0) (v3[2])which implies that v3[1] is free and v3[2] = 0. Therefore, v3 = (1, 0).b. Find the most general real-valued solution to the linear system of differential equations. Use t as the independent variable in your answers.The general solution of the system is given byy(t) = c1 e^(0 t) (0, 1) + c2 e^(13t) (1, 0) + c3 (t e^(0 t) (0, 1) + e^(0 t) (1, 0))Therefore, the most general real-valued solution to the linear system of differential equations is given byy(t) = c1(0, 1) + c2 e^(13t)(1, 0) + c3 (t(0, 1) + (1, 0))c. Solve the original initial value problem.To solve the given initial value problem, we need to determine the coefficients c1, c2, and c3 that satisfy the initial conditions:y(0) = a = c1(0, 1) + c2(1, 0) + c3(0, 1) + (1, 0)c1 = 0, c2 = a - 1, and c3 = 0Therefore, the solution to the initial value problem is given byy(t) = (a - 1) e^(13t) (1, 0) + (1, 0) = [a e^(13t) - (e^(13t) - 1)] (1, 0) = y1(t)The solution to the initial value problem is yı(t) = ya(t) = [a e^(13t) - (e^(13t) - 1)] (1, 0).

Learn more about Initial

brainly.com/question/13243199

#SPJ11

Mei Mei has a part-time job at an ice skating rink selling hot cocoa. She decided to plot the number of hot cocoas she sold relative to the day's high temperature and then draw the line of best fit. Based on the line of best fit, how many hot cocoas would you predict Mei Mei to sell if the day’s high temperature were 42^{\circ} ∘ F?

Answers

The following table shows how many hot cocoas Mei Mei would likely sell if the day's high temperature was 42 oF:

58.

How is a linear function defined?

The following is the definition of a linear function's slope and intercept:

y = mx + b.

The slope, or m, indicates the rate of change.

The value of y at x=0 is represented by the intercept, b.

At y = 100, where the function meets the y-axis, the intercept b is defined as follows:

b = 100.

As y decays to 94 when x rises to six, the slope m is calculated as follows:

m = (94 - 100)/(6 - 0) (6 - 0)

m = -1.

As a result, the function that forecasts how much hot chocolate will be sold at a high temperature.

Lack of Information

The graphic provided at the end of the response provides the linear function's graph.

To know more about linear function, click the below link

https://brainly.com/question/20286983

#SPJ4

Make up a sequences that have (a) 3,3,3,3,... as its second differences. (b) 1, 2,3,4,5,... as its third differences (c) 1, 2, 4,8,16,... as its 100th differences.

Answers

The nth term of the sequence is 2^n.

(a) 3, 3, 3, 3, ... is a sequence that has 0 for both its first and second differences. That is, every term in the sequence is the same.(b) The sequence is the series of natural numbers. It has 0 for its first and second differences, and 6 for its third differences. The nth term of the sequence is n.(c) The sequence has 0 for its first 99 differences and 100! for its 100th difference. The nth term of the sequence is 2^n.

Learn more about Sequence

brainly.com/question/1606838

#SPJ11

Edith's van can safely carry a maximum land of 920 kilograms.

She wants to use her van to carry

30 sacks of potatoes, each of mass 25 kilograms to the nearest kilogram

and

20 sacks of carots, each of mass 7. 5 kilograms to 1 decimal place

Can she definitely use her van safety in one journey?

You must show your working

(4 marks)

Answers

Yes, she can definitely use her van safety in one journey. The total weight is the weight of potatoes plus the weight of carots.

Given,

Number of sacks of potatoes = 30

Mass of each potato = 25

Total weight of Potatoes = 30 * 25

= 750

Number of sacks of carrots = 20

Mass of each Carot = 7.5

Total weight of Carrot = 20 * 7.5

= 150

(The total weight is the weight of potatoes plus the weight of carrots.)

So, total weight = 750 + 150

= 900

The maximum land which Edith can carry safely in the van is 920 kilograms.

∵ 900 < 920

∴ She can use her van safely.

Learn more about Weight here: brainly.com/question/23312072

#SPJ4

Enter the correct answer in the box
Function is shown on the graph in function is the parent quadratic function what is the equation of transtormed function in terms of ?

Answers

In quadratic functiοn -

          g(x)                                    h(x)                                     d(x)

vertical shift dοwn 3     reflectiοn acrοss the x-axis   vertical shift dοwn 3

hοrizοntal shift left 3     vertical strecht οf 3               hοrizοntal shift right 3

What is the οperatiοn οf a quadratic functiοn?

The expressiοn f(x) = ax² + bx + c, where a, b, and c are all integers with a nοt equal tο zerο, denοtes a quadratic functiοn. The shape that a quadratic functiοn's graph takes οn is called a parabοla.

In spite οf the fact that parabοlas might have οpenings that are upward οr dοwnward, varying "widths" οr "steepnesses," they all have the same basic "U" shape.

Quadratic parent: f(x)=x²

The graph is a parabοla with vertex V=(0,0) at the οrigin and οpens up.

When x=1→f(1)=1²→f(1)=1

1) g(x)

The graph οpens up, then there is nοt a reflectiοn acrοss the x-axis.

The vertex is at the pοint (-3,-3): 3 units tο the left and 3 units dοwn οf the vertex οf the parent funtiοn.

When x is 1 unit tο the right frοm the vertex g(x)=1

Then the transfοrmatiοns were applied tο the cuadratic parent functiοn are:

1.1) vertical shift dοwn 3.

1.2) hοrizοntal shift left 3.

2) h(x)

The graph οpens dοwn, then there is a reflectiοn acrοss the x-axis.

The vertex is at the οrigin (0,0).

When x is 1 unit tο the right frοm the vertex h(x)=-3

Then the transfοrmatiοns were applied tο the quadratic parent functiοn are:

2.1) reflectiοn acrοss the x-axis.

2.2) vertical strecht οf 3.

3) d(x)

The graph οpens up, then there is nοt a reflectiοn acrοss the x-axis.

The vertex is at the pοint (3,-3): 3 units tο the right and 3 units dοwn οf the vertex οf the parent funtiοn.

When x is 1 unit tο the right frοm the vertex d(x)=1

Then the transfοrmatiοns were applied tο the cuadratic parent functiοn are:

3.1) vertical shift dοwn 3.

3.2) hοrizοntal shift right 3.

Learn more about quadratic function

brainly.com/question/30929439

#SPJ1

The complete question is -

The given graphs show functions which have been transformed from the quadratic parent, f(x) = x2. Determine which transformations were applied to the quadratic parent function to result in each graph.

which sampling approach was used in the following statement?kane randomly sampled 250 nurses from urban areas and 250 from rural areas from a list of licensed nurses in wisconsin to study their attitudes toward evidence-based practice.

Answers

The sampling approach that was used in the statement "Kane randomly sampled 250 nurses from urban areas and 250 from rural areas from a list of licensed nurses in Wisconsin to study their attitudes toward evidence-based practice" is Stratified random sampling.

What is Stratified random sampling?

Stratified random sampling is a method of sampling that is based on dividing the population into subgroups called strata. Stratified random sampling is a statistical sampling method that involves the division of the population into subgroups or strata, and a sample is then drawn from each stratum in proportion to the size of the stratum. It's a sampling method that ensures the representation of all population strata in the sample, making it more effective than simple random sampling.

Stratified random sampling is used when there are variations in the population that are likely to influence the outcome of the study. The stratified random sampling method is used to ensure that these differences are reflected in the sample. In this way, the results of the study are more representative of the entire population than they would be if a simple random sample were used.

Learn more about Stratified random sampling here: https://brainly.com/question/20544692.

#SPJ11

Bella is splitting her rectangular backyard into a garden in the shape of a trapezoid and a fish pond in the shape of a right triangle. What is the area of her garden?

Answers

The Area of Bella's garden as required to be determined in the task content is the difference of the area of the rectangular backyard and the right triangular fish pond.

What is the area of Bella's trapezoidal garden?

It follows from the task content that the area of Bella's trapezoidal garden is to be determined from the given information.

Since the garden and the fish pond are from the rectangular backyard; the sum of their areas is equal to the area of the backyard.

Ultimately, the area of the garden is the difference of the area of the rectangular backyard and the right triangular fish pond.

Read more on difference;

https://brainly.com/question/28368605

$SPJ1

are the statements equivalent? "the number x is greater than 5 by 3" and x -3=5

Answers

Answer:

x=8

Step-by-step explanation:

x-3=5

+3=+3(cancel 3)

_____________

x=8

Find the derivative of the function f(x), below. It may be to your advantage to simplify before differentiating. f(x)=ln(14-e^-2x). f'(x)

Answers


To find the derivative of the function f(x), we need to take the derivative of the natural logarithm (ln) of the function. We can do this by using the chain rule, which states that the derivative of the composition of two functions is equal to the derivative of the outer function times the derivative of the inner function.

The derivative of the outer function (ln) is 1/f(x), and the derivative of the inner function (14 - e-2x) is -2e-2x. So the derivative of f(x) is:

f'(x) = 1/f(x) × (-2e-2x)

f'(x) = 1/(ln(14 - e-2x)) × (-2e-2x)

f'(x) = -2e-2x/(14 - e-2x)

To learn more about “derivatives” refer to the : https://brainly.com/question/12047216

#SPJ11

a careless university student leaves her iclicker device behind with probability 1/4 each time she attends a class. she sets out with her iclicker device to attend 5 different classes (each class is in a different lecture theatre). part 1) if she arrives home without her iclicker device and she is sure she has the iclicker device after leaving the first class, what is the probability (to 3 significant figures) that she left it in the 5th class? probability

Answers

The probability of leaving it in any given class is 1/4, the probability of leaving it in the 5th class is 1/4, or[tex]0.250 (25.0%).[/tex]

The probability that the student left her iClicker device in the 5th class is 0.250 (25.0%). This is because the probability of leaving it in any given class is 1/4, and as she attends 5 different classes,

there is 5/4 or 1.25 chances of her leaving it in the 5th class.

To explain further, the probability of an event can be calculated by dividing the number of ways the event can occur by the total number of possible outcomes. In this case, the event is leaving the iClicker device in the 5th class, and the total number of possible outcomes is 5 (for the 5 classes).

for such more questions on probability

https://brainly.com/question/24756209

#SPJ11

A triangle has two sides of length 3 and 16. What is the largest possible whole-number length for the third side

Answers

The largest possible whole-number length for the third side is 18, which satisfies all three inequalities.

What is inequality theorem?

The triangle inequality theorem explains the relationship between the three sides of a triangle. This theorem states that for any triangle, the sum of the lengths of the first two sides is always larger than the length of the third side.

According to question:

Let x be the length of the third side. By the triangle inequality, we have:

3 + 16 > x and 16 + x > 3 and 3 + x > 16

Simplifying, we get:

19 > x and x > 13 and x < 19

The largest possible whole-number length for the third side is 18, which satisfies all three inequalities.

To know more about Inequality visit:

brainly.com/question/30228778

#SPJ1

3. Assume that a simple random sample has been selected from a normally distributed population. Find the test statistic, P-value, critical value(s), and state the final conclusion.
Test the claim that the mean lifetime of car engines of a particular type is greater than 220,000 miles. Sample data are summarized as n = 23.x=226,450 miles, and s- 11,500 miles. Use a significance
a=0.01. Select the correct test statistic and critical value.
Test statistic: t 2.6898. Critical value: t= -2.508.
Test statistic: t=2.6898. Critical value: t = 2.508.
Test statistic: t = -2.6898. Critical value: t = 12.508.
All change
Test statistic: z = 2.6898. Critical value: z = +2.508.

Answers

There is sufficient evidence tο say that the mean lifetime οf car engines οf a particular type is greater than 220,000 miles.

What is Hypothesis Test?

The cοnclusiοn οf the hypοthesis test shοuld be the same whether using the critical value apprοach οr using the p-value apprοach. Mοst οf the time the p-value is used tο test a hypοthesis and it is alsο knοwn as the mοdern apprοach.

Hο: μ ≤ 220000

Ha: μ > 220000

Upper Tail

The significance level,

α = 0.01

The sample mean,

¯x = 226450

The sample standard deviation,

s = 11500

The sample size,

n=23

Test statistics:

[tex]$t={\frac{{\bar{x}}-\mu}{{\frac{s}{\sqrt{n}}}}}$[/tex]

[tex]$={\frac{226450-2220000}{\dfrac{11500}{\sqrt{23} y} }$[/tex]

≈ 2.69

Decision rule:

Reject the null if,

p-value ≤α

Decision rule:

Reject the null if,

|Calculated t |≥|Critical t|

Degrees of freedom:

Df = n−1

=22

We can use a t-distribution table, an online tool, or the following excel command to calculate the p-value using excel =TDIST(2.69, 22, 1):

p-value = P(t>2.69)

≈0.0067

Critical value of t using excel: =TINV(0.02, 22)

t(critical) = tα,

df = t(0.01,22)

≈2.51

Decision:

p-value ≈0.0067<α =0.0100

Reject the null.

Conclusion:

There is sufficient evidence to say that the mean lifetime of car engines of a particular type is greater than 220,000 miles.

To know more about test statistic visit:

brainly.com/question/29296846

#SPJ1

Complete question:

Simplify the product: (4x-3) (3x+1)
O A. x-2
OB. 12x²-3
O C. 12x-5x-3
OD. 12x³-9x² +4x-3

Answers

Answer:

[tex]\large\boxed{\mathtt{C) \ 12x^{2}-5x-3}}[/tex]

Step-by-step explanation:

[tex]\textsf{For this problem, we are asked to multiply 2 Binomials.}[/tex]

[tex]\textsf{We should use the FOIL Method.}[/tex]

[tex]\large\underline{\textsf{FOIL Broken Down;}}[/tex]

[tex]\textsf{F - Fronts}[/tex]

[tex]\textsf{O - Outers}[/tex]

[tex]\textsf{I - Inners}[/tex]

[tex]\textsf{L - Last}[/tex]

[tex]\large\underline{\textsf{For Example.}}[/tex]

[tex]\mathtt{(2x+2y)(3x+3y)}[/tex]

[tex]\mathtt{(F) \ 2x \times 3x = 6x^{2}}[/tex]

[tex]\mathtt{(O) \ 2x \times 3y = 6xy}[/tex]

[tex]\mathtt{(I) \ 2y \times 3x = 6xy}[/tex]

[tex]\mathtt{(L) \ 2y \times 3y = 6y^{2}}[/tex]

[tex]\textsf{Let's use the FOIL Method for our problem.}[/tex]

[tex]\large\underline{\textsf{FOIL;}}[/tex]

[tex]\mathtt{(4x-3)(3x+1)}[/tex]

[tex]\mathtt{(F) \ 4x \times 3x = 12x^{2}}[/tex]

[tex]\mathtt{(O) \ 4x \times 1 = 4x}[/tex]

[tex]\mathtt{(I) \ -3 \times 3x = -9x}[/tex]

[tex]\mathtt{(L) \ -3 \times 1 = -3}[/tex]

[tex]\large\underline{\textsf{We should have;}}[/tex]

[tex]\mathtt{12x^{2}+4x-9x-3}[/tex]

[tex]\large\underline{\textsf{Combine Like Terms;}}[/tex]

[tex]\large\boxed{\mathtt{C) \ 12x^{2}-5x-3}}[/tex]

Select all of the reasons that the range is an appropriate measure of variability to describe the variation in the hours slept by eighth graders.

A double box plot showing hours sleeping. For seventh graders, the left whisker is at 7.5, the left edge of the box is at eight, the line inside the box is at 8.5, the right edge of the box is at nine, and the right whisker is at 9.5. For eighth graders, the left whisker is at seven, the left end of the box is at 7.5, the line inside the box is at eight, the right end of the box is at 8.5, and the right whisker is at nine. Screen reader support enabled.

Answers

All of the reasons that the range is an appropriate measure of variability to describe the variation in the hours slept by eighth graders include the following:

A. The data are evenly distributed between 7 hours and 9 hours.

D. There is no outlier in the data.

What is a range?

In Mathematics, a range can be defined as the difference between the highest number and the lowest number contained in a data set.

Mathematically, the range of a data set can be calculated by using the following mathematical equation;

Range = Highest number - Lowest number

By critically observing the double box-and-whisker plots or box plot for the variation in the hours slept by eighth graders, we can logically deduce that there is no outlier in the data because they are evenly distributed and centered about the mean.

In conclusion, the box-and-whisker plot or box plot is symmetrical.

Read more on boxplot here: brainly.com/question/29648407

#SPJ1

Answer:

Step-by-step explanation:

Yeah what the other guy said

a manager recorded the number of gallons of ice cream sold for the past six periods. he asked you to choose a forecasting model to predict the demand for gallons of ice cream in period 7. you consider applying a two-period moving average model and a two-period weighted moving average model with weights of 0.6 and 0.4. a) which model is better for this data set (hint: show all your work including forecasts for each period and calculations using measures of forecast accuracy)? (9 points)

Answers

The two-period moving average model and the two-period weighted moving average model are both common forecasting methods used to predict future demand. and we understand that the model with the lower MAD and MSE values will have the most accurate forecast.

To determine which model is better for this particular data set, we need to compare the accuracy of each model. To do this, we will calculate the Mean Absolute Deviation (MAD) and the Mean Squared Error (MSE) for each model.
For the two-period moving average model, we can calculate the forecast for period 7 by taking the average of p5 and 6:

Period 7 forecast = (Gallons in Period 5 + Gallons in Period 6)/2

For the two-period weighted moving average model, we can calculate the forecast for period 7 by using the weights of 0.6 and 0.4:

Period 7 forecast = (0.6 x Gallons in Period 5) + (0.4 x Gallons in Period 6)

We can then compare the accuracy of each model by calculating the MAD and MSE. To calculate MAD, we need to subtract the actual demand in each period from the forecasted demand and take the absolute value:

MAD = |Actual demand – Forecasted demand|

To calculate MSE, we need to square the differences between the actual demand and the forecasted demand:

MSE = (Actual demand – Forecasted demand)^2

After calculating the MAD and MSE for each model, we can compare the results to determine which model is better for this data set. The model with the lower MAD and MSE values will have the most accurate forecast.

For more such questions on forecast

https://brainly.com/question/24730207

#SPJ11

She begins at sea level, which is an elevation of 0 feet.
She descends for 50 seconds at a speed of 5 feet per second.
She then ascends for 54 seconds at a speed of 4.4 feet per second.

Answers

Answer:

The diver descends:

50 seconds x 5 feet/second = 250 feet

The diver ascends:

54 seconds x 4.4 feet/second = 237.6 feet

Therefore, the total change in elevation is:

250 feet (descent) - 237.6 feet (ascent) = 12.4 feet

So, the diver's final elevation is:

0 feet (starting elevation) - 12.4 feet (change in elevation) = -12.4 feet

Therefore, the diver ends up 12.4 feet below sea level.

Answer:

it is 1

Step-by-step explanation:

its is 1 2 3 hsvs jafsnsjhd jsusgsmsi jshsbjdg

1. Use least-squares regression to fit a straight line to x 6 7 11 15 17 21 23 29 29 37 39 y 29 21 29 14 21 15 7 7 13 0 3 Along with the slope and the intercept, compute the standard error of the estimate and the correlation coefficient. Please write a detailed solution with explanation

Answers

The least-squares regression analysis can provide important information about the relationship between two variables, including the slope and intercept of the fitted line, the standard error of the estimate, and the correlation coefficient.

What are the steps to use least-squares regression?

To use least-squares regression to fit a straight line to the given data points, follow these steps:

Calculate the means of x and y values:
  x_mean = (6 + 7 + 11 + 15 + 17 + 21 + 23 + 29 + 29 + 37 + 39) / 11 = 21.18
  y_mean = (29 + 21 + 29 + 14 + 21 + 15 + 7 + 7 + 13 + 0 + 3) / 11 = 14.55

Calculate the sums of squared differences for x and y:
  Σ(x-x_mean)^2 = Σ(y-y_mean)² = Σ(x-x_mean)(y-y_mean) = 0
  (Compute these sums using the given x and y values)

Calculate the slope (b) of the fitted straight line:
  b = Σ(x-x_mean)(y-y_mean) / Σ(x-x_mean)²

Calculate the intercept (a) of the fitted straight line:
  a = y_mean - b * x_mean
Calculate the standard error of the estimate (SEE):
  SEE = sqrt(Σ(y-y_pred)² / (n-2))
  (y_pred represents the predicted y values using the fitted straight line equation)

Calculate the correlation coefficient (r):
  r = Σ(x-x_mean)(y-y_mean) / sqrt(Σ(x-x_mean)² x  Σ(y-y_mean)²

Once you have followed these steps using the given x and y values, you will have the least-squares regression line, along with the slope and intercept, the standard error of the estimate, and the correlation coefficient.

Learn more about least-squares regression

brainly.com/question/30044885

#SPJ11

Other Questions
Tyler is a massage therapist who notices a strange yellowish cast to his client Amy's skin. Amy has been his client for a long time, and he's neverseen her skin look like this before. He asks Amy about it, and she says it's kind of weird, but she's not that worried about it because she thinksshe's just a little under the weather. What would be the BEST way for Tyler to respond to his client? the law of one price is best described as: the true fundamental value of an asset. earning a risk-free profit without committing any capital. two assets that will produce the same cash flows in the future must sell for equivalent prices. which event triggers the creation of an action potential? Eckhart School is selling candy to raise money for its sports programs. Last year the school raised $480 during a 12-day fundraiser. The school expects this year's daily sales to be in proportion to last year's daily sales. Based on the school's expectations, which of the following statements is correct?A) In 3 days, the school will raise $120.B) In 7 days, the school will raise $240.C) In 13 days, the school will raise $500.D) In 22 days, the school will raise $960. The magnitudes of the current density and the diameters for wires 1 and 2 are given in the table. The current directions are indicated by the arrows.Find the current I3 in wire 3.Express your answer in amperes to two significant figures. Call current out of the junction positive and current into the junction negative.The magnitudes of the current density and the diamWireCurrent density(A/mm2)Diameter(mm)12.11.723.92.4 Some behaviors such as mating and caring for young are geneticallydetermined in certain species of birds. The presence of these behaviors ismost likely due to the fact thatA) birds do not have the ability to learnB) individual birds need to learn to survive and reproduceC) these behaviors helped birds to survive in the pastD) within their lifetimes, birds developed these behaviors XYZ company is situated in Ghana. They have been commissioned your organisation to design a database for them. The database is expected to keep data on employees, customers, suppliers, and products. Important records on employees such as employee's ID, date of birth, and dependants are expected to be captured in the database. Products information such as product's ID, name of product, manufacturing and expiring data, and name of supplier are expected to be captured. The company receives suppliers from different organisations, hence, it would like the database to capture relevant details of these suppliers. Each supplier supplies only one type of product for the company. Every customer is assigned one sales representative, yet sales representatives maybe assigned up to ten customers. Customers can order an unlimited number of good. Properly represent all entities, relationships, constraints, and appropriate keys in an E-R diagram that can readily be used in a database. Which of the following is a method used in the slip-critical bolted connections to make sure bolts are properly tensioned?turn of nut methoddirect tension indicatorcalibrated torque wrench methodsplined tension control bolts I'm struggling to find the rhetorical inquiries of the poem "Thoughts of Hanoi" by Nguyen Thi Vinh Can someone please help me distunguish between a nucleotide and a nucleic acid in the form of a diagram (structure) urgently please Thanks :) explain collateralized debt obligations (cdos). a cdo is a package of debt securities that are usually separated into tranches based on their initial price. cdos are packages that contain only mortgage types of debt securities. a cdo represents a package of debt securities backed by collateral that is sold to investors. If you know that a < b, and both a and b are positive numbers, then what must be true about the relationship between the opposites of these numbers? Explain. Frederick Taylor, often referred to as the father of scientific management, is best known for his work withA. time-motion studies.B. the Hawthorne studies.C. the hierarchy of needs.D. Theory X.E. employee punishment. a cup of hot coffee is placed outside where the temperature is 0, assume the coffee cools to approach the outside temperature according to an exponential decay model, if the continuous rate of cooling is determined to be 2 percent per minute and the current temperature of the coffee is 54.8 celsius how many minutes will the coffee cool to 44.9 Celsius The authors U.S. constitution included which two features as ways to limit the power of any one person in the government On a Linux system using RPM for package management, which of the following commands would you use to search online repositories for an available Firefox package?a. apt search firefoxb. yum search firefoxc. rpm -s firefoxd. apt-get search firefox When driving in normal conditions on a rural roadway, your following distance should be a. no more than one second. b. three seconds. c. twelve seconds. Write 0. 0166 correct to two significant figures. SynXis Property Hub isChoose all that apply, and then select Submit.A) A fully cloud based system.B) Compatible with most devices.C) Seamlessly integrated with SynXis CR.D) Only compatible with Windows 7 workstations.SUBMIT4 russell is doing some research before buying his first house. he is looking at two different areas of the city, and he wants to know if there is a significant difference between the mean prices of homes in the two areas. for the 40 homes he samples in the first area, the mean home price is $183,100. public records indicate that home prices in the first area have a population standard deviation of $21,845. for the 33 homes he samples in the second area, the mean home price is $161,500. again, public records show that home prices in the second area have a population standard deviation of $24,820. let population 1 be homes in the first area and population 2 be homes in the second area. construct a 95% confidence interval for the true difference between the mean home prices in the two areas. round the endpoints of the interval to the nearest whole number, if necessary.